Are there continuous functions who are the same in an interval but differ in at least one other point? Announcing the arrival of Valued Associate #679: Cesar Manara Planned maintenance scheduled April 23, 2019 at 23:30 UTC (7:30pm US/Eastern)Let $f:[a,infty] rightarrow mathbb R$ and $lim_xrightarrowinfty f'(x) = infty$, show that $f$ is not uniformly continuousFunction which is continuous everywhere in its domain, but differentiable only at one pointContinuous functions that pointwise converge to zero, but the integral tending towards infinity.Is every such function convex or concave?Maximum diameter for the preimage of a point when the degree is not 1 or -1Prove that on every great circle on the earth there are antipodal points at which the temperature is the same.Is there a function on a compact interval that is differentiable but not Lipschitz continuous?Counter-example for continuous functionNowhere continuous function for every equivalence classPeriodic function with period $T$; show that there exists a point in the interval with $f(x) = f(x + T /2).$

Sally's older brother

How do you cope with tons of web fonts when copying and pasting from web pages?

Did John Wesley plagiarize Matthew Henry...?

3D Masyu - A Die

Understanding piped commands in GNU/Linux

Did pre-Columbian Americans know the spherical shape of the Earth?

How to ask rejected full-time candidates to apply to teach individual courses?

Vertical ranges of Column Plots in 12

Table formatting with tabularx?

Why did Bronn offer to be Tyrion Lannister's champion in trial by combat?

What did Turing mean when saying that "machines cannot give rise to surprises" is due to a fallacy?

The Nth Gryphon Number

What helicopter has the most rotor blades?

How could a hydrazine and N2O4 cloud (or it's reactants) show up in weather radar?

As a dual citizen, my US passport will expire one day after traveling to the US. Will this work?

Does the universe have a fixed centre of mass?

My mentor says to set image to Fine instead of RAW — how is this different from JPG?

Simple Line in LaTeX Help!

What should one know about term logic before studying propositional and predicate logic?

How do I say "this must not happen"?

Can I cut the hair of a conjured korred with a blade made of precious material to harvest that material from the korred?

Getting representations of the Lie group out of representations of its Lie algebra

Is the time—manner—place ordering of adverbials an oversimplification?

How can I prevent/balance waiting and turtling as a response to cooldown mechanics



Are there continuous functions who are the same in an interval but differ in at least one other point?



Announcing the arrival of Valued Associate #679: Cesar Manara
Planned maintenance scheduled April 23, 2019 at 23:30 UTC (7:30pm US/Eastern)Let $f:[a,infty] rightarrow mathbb R$ and $lim_xrightarrowinfty f'(x) = infty$, show that $f$ is not uniformly continuousFunction which is continuous everywhere in its domain, but differentiable only at one pointContinuous functions that pointwise converge to zero, but the integral tending towards infinity.Is every such function convex or concave?Maximum diameter for the preimage of a point when the degree is not 1 or -1Prove that on every great circle on the earth there are antipodal points at which the temperature is the same.Is there a function on a compact interval that is differentiable but not Lipschitz continuous?Counter-example for continuous functionNowhere continuous function for every equivalence classPeriodic function with period $T$; show that there exists a point in the interval with $f(x) = f(x + T /2).$










12












$begingroup$


You are given a function $f: mathbbRrightarrow mathbbR$. Every derivative $fracd^ndx^n(f(x)), ,n >0$ of the function is continuous.



Is there a function $g: mathbbRrightarrow mathbbR$, for which every derivative $fracd^ndx^n(g(x)), ,n >0$ is also continuous, such that:
$$forall xin[a,b]: , g(x) = f(x)land , exists x notin [a,b]: f(x) neq g(x),, a neq b$$



Thanks!



Edit:



I asked the question because I intuitively wondered if there would be functions, which could behave like "the same" in a given interval, but then behave differently so that they start diverging or at least stopped being the same the anymore. The answer to this question gave me a bigger understanding of real analysis.



If you would like to know which made me think about such a problem:
Although this is a vague formulation, generally, this question asks if two (completely) different things can develop (themselves) to be exactly equal in at least one part of there whole existence...










share|cite|improve this question











$endgroup$







  • 3




    $begingroup$
    FYI, this can't happen with complex-differentiable $f: mathbbC to mathbbC$. Such functions are immediately infinitely often differentiable, so it indeed matches your setting. If two complex-differentiable functions agree on an open set, they already agree everywhere on their domain.
    $endgroup$
    – ComFreek
    Apr 13 at 13:58











  • $begingroup$
    How to proof that?
    $endgroup$
    – TVSuchty
    Apr 13 at 17:28






  • 3




    $begingroup$
    The proof of the so-called "identity theorem" is usually based on several previous lemmas. I think this comment section is too small for that, but I suggest having a look at identity theorem @ Wikipedia or in any introductory complex analysis book.
    $endgroup$
    – ComFreek
    Apr 15 at 6:35







  • 1




    $begingroup$
    Why is this question still on hold?
    $endgroup$
    – TVSuchty
    20 hours ago










  • $begingroup$
    I think that's a perfectly fine and on-topic question. I asked in chat about it. I'd suggest opening a question on meta if nobody responds in chat about it.
    $endgroup$
    – ComFreek
    14 hours ago















12












$begingroup$


You are given a function $f: mathbbRrightarrow mathbbR$. Every derivative $fracd^ndx^n(f(x)), ,n >0$ of the function is continuous.



Is there a function $g: mathbbRrightarrow mathbbR$, for which every derivative $fracd^ndx^n(g(x)), ,n >0$ is also continuous, such that:
$$forall xin[a,b]: , g(x) = f(x)land , exists x notin [a,b]: f(x) neq g(x),, a neq b$$



Thanks!



Edit:



I asked the question because I intuitively wondered if there would be functions, which could behave like "the same" in a given interval, but then behave differently so that they start diverging or at least stopped being the same the anymore. The answer to this question gave me a bigger understanding of real analysis.



If you would like to know which made me think about such a problem:
Although this is a vague formulation, generally, this question asks if two (completely) different things can develop (themselves) to be exactly equal in at least one part of there whole existence...










share|cite|improve this question











$endgroup$







  • 3




    $begingroup$
    FYI, this can't happen with complex-differentiable $f: mathbbC to mathbbC$. Such functions are immediately infinitely often differentiable, so it indeed matches your setting. If two complex-differentiable functions agree on an open set, they already agree everywhere on their domain.
    $endgroup$
    – ComFreek
    Apr 13 at 13:58











  • $begingroup$
    How to proof that?
    $endgroup$
    – TVSuchty
    Apr 13 at 17:28






  • 3




    $begingroup$
    The proof of the so-called "identity theorem" is usually based on several previous lemmas. I think this comment section is too small for that, but I suggest having a look at identity theorem @ Wikipedia or in any introductory complex analysis book.
    $endgroup$
    – ComFreek
    Apr 15 at 6:35







  • 1




    $begingroup$
    Why is this question still on hold?
    $endgroup$
    – TVSuchty
    20 hours ago










  • $begingroup$
    I think that's a perfectly fine and on-topic question. I asked in chat about it. I'd suggest opening a question on meta if nobody responds in chat about it.
    $endgroup$
    – ComFreek
    14 hours ago













12












12








12


2



$begingroup$


You are given a function $f: mathbbRrightarrow mathbbR$. Every derivative $fracd^ndx^n(f(x)), ,n >0$ of the function is continuous.



Is there a function $g: mathbbRrightarrow mathbbR$, for which every derivative $fracd^ndx^n(g(x)), ,n >0$ is also continuous, such that:
$$forall xin[a,b]: , g(x) = f(x)land , exists x notin [a,b]: f(x) neq g(x),, a neq b$$



Thanks!



Edit:



I asked the question because I intuitively wondered if there would be functions, which could behave like "the same" in a given interval, but then behave differently so that they start diverging or at least stopped being the same the anymore. The answer to this question gave me a bigger understanding of real analysis.



If you would like to know which made me think about such a problem:
Although this is a vague formulation, generally, this question asks if two (completely) different things can develop (themselves) to be exactly equal in at least one part of there whole existence...










share|cite|improve this question











$endgroup$




You are given a function $f: mathbbRrightarrow mathbbR$. Every derivative $fracd^ndx^n(f(x)), ,n >0$ of the function is continuous.



Is there a function $g: mathbbRrightarrow mathbbR$, for which every derivative $fracd^ndx^n(g(x)), ,n >0$ is also continuous, such that:
$$forall xin[a,b]: , g(x) = f(x)land , exists x notin [a,b]: f(x) neq g(x),, a neq b$$



Thanks!



Edit:



I asked the question because I intuitively wondered if there would be functions, which could behave like "the same" in a given interval, but then behave differently so that they start diverging or at least stopped being the same the anymore. The answer to this question gave me a bigger understanding of real analysis.



If you would like to know which made me think about such a problem:
Although this is a vague formulation, generally, this question asks if two (completely) different things can develop (themselves) to be exactly equal in at least one part of there whole existence...







real-analysis calculus






share|cite|improve this question















share|cite|improve this question













share|cite|improve this question




share|cite|improve this question








edited yesterday







TVSuchty

















asked Apr 12 at 22:21









TVSuchtyTVSuchty

1056




1056







  • 3




    $begingroup$
    FYI, this can't happen with complex-differentiable $f: mathbbC to mathbbC$. Such functions are immediately infinitely often differentiable, so it indeed matches your setting. If two complex-differentiable functions agree on an open set, they already agree everywhere on their domain.
    $endgroup$
    – ComFreek
    Apr 13 at 13:58











  • $begingroup$
    How to proof that?
    $endgroup$
    – TVSuchty
    Apr 13 at 17:28






  • 3




    $begingroup$
    The proof of the so-called "identity theorem" is usually based on several previous lemmas. I think this comment section is too small for that, but I suggest having a look at identity theorem @ Wikipedia or in any introductory complex analysis book.
    $endgroup$
    – ComFreek
    Apr 15 at 6:35







  • 1




    $begingroup$
    Why is this question still on hold?
    $endgroup$
    – TVSuchty
    20 hours ago










  • $begingroup$
    I think that's a perfectly fine and on-topic question. I asked in chat about it. I'd suggest opening a question on meta if nobody responds in chat about it.
    $endgroup$
    – ComFreek
    14 hours ago












  • 3




    $begingroup$
    FYI, this can't happen with complex-differentiable $f: mathbbC to mathbbC$. Such functions are immediately infinitely often differentiable, so it indeed matches your setting. If two complex-differentiable functions agree on an open set, they already agree everywhere on their domain.
    $endgroup$
    – ComFreek
    Apr 13 at 13:58











  • $begingroup$
    How to proof that?
    $endgroup$
    – TVSuchty
    Apr 13 at 17:28






  • 3




    $begingroup$
    The proof of the so-called "identity theorem" is usually based on several previous lemmas. I think this comment section is too small for that, but I suggest having a look at identity theorem @ Wikipedia or in any introductory complex analysis book.
    $endgroup$
    – ComFreek
    Apr 15 at 6:35







  • 1




    $begingroup$
    Why is this question still on hold?
    $endgroup$
    – TVSuchty
    20 hours ago










  • $begingroup$
    I think that's a perfectly fine and on-topic question. I asked in chat about it. I'd suggest opening a question on meta if nobody responds in chat about it.
    $endgroup$
    – ComFreek
    14 hours ago







3




3




$begingroup$
FYI, this can't happen with complex-differentiable $f: mathbbC to mathbbC$. Such functions are immediately infinitely often differentiable, so it indeed matches your setting. If two complex-differentiable functions agree on an open set, they already agree everywhere on their domain.
$endgroup$
– ComFreek
Apr 13 at 13:58





$begingroup$
FYI, this can't happen with complex-differentiable $f: mathbbC to mathbbC$. Such functions are immediately infinitely often differentiable, so it indeed matches your setting. If two complex-differentiable functions agree on an open set, they already agree everywhere on their domain.
$endgroup$
– ComFreek
Apr 13 at 13:58













$begingroup$
How to proof that?
$endgroup$
– TVSuchty
Apr 13 at 17:28




$begingroup$
How to proof that?
$endgroup$
– TVSuchty
Apr 13 at 17:28




3




3




$begingroup$
The proof of the so-called "identity theorem" is usually based on several previous lemmas. I think this comment section is too small for that, but I suggest having a look at identity theorem @ Wikipedia or in any introductory complex analysis book.
$endgroup$
– ComFreek
Apr 15 at 6:35





$begingroup$
The proof of the so-called "identity theorem" is usually based on several previous lemmas. I think this comment section is too small for that, but I suggest having a look at identity theorem @ Wikipedia or in any introductory complex analysis book.
$endgroup$
– ComFreek
Apr 15 at 6:35





1




1




$begingroup$
Why is this question still on hold?
$endgroup$
– TVSuchty
20 hours ago




$begingroup$
Why is this question still on hold?
$endgroup$
– TVSuchty
20 hours ago












$begingroup$
I think that's a perfectly fine and on-topic question. I asked in chat about it. I'd suggest opening a question on meta if nobody responds in chat about it.
$endgroup$
– ComFreek
14 hours ago




$begingroup$
I think that's a perfectly fine and on-topic question. I asked in chat about it. I'd suggest opening a question on meta if nobody responds in chat about it.
$endgroup$
– ComFreek
14 hours ago










2 Answers
2






active

oldest

votes


















18












$begingroup$

Define the real functions $f$ and $g$ thus:
$$
f(x) = begincases expBig(-frac1(x - 1)^2Big) &textif x > 1 \
0 &textif x in [-1, 1] \
expBig(-frac1(x + 1)^2Big) &textif x < -1
endcases
$$
and
$g(x) = 0$. $f$ and $g$ are both $0$ on $[-1, 1]$ but they differ in value everywhere else.



Obviously $g$ is continuously differentiable infinitely many times as it is a constant function. You can also check that $f$ is continuously differentiable infinitely many times at $x = -1$ and $x = 1$ by applying L'Hôpital's rule inductively (Edit: A more elementary demonstration is given below based on the exponential series). Checking this is a fine exercise in Real Analysis; you should try it. Here is a first taste of it:
beginalign*
lim_x to 1^+fracdf(x)dx &= limlimits_x to 1^+frac2expbig(- frac1(x - 1)^2big)(x - 1)^3 \
&= 2lim_x to 1^+fracfrac1(x - 1)^3expbig(frac1(x - 1)^2big) quadtextthis limit is of the form fracinftyinfty text so L'Hôpital applies \
&= 2 lim_x to 1^+fracfracddx(x - 1)^-3fracddxexpbig(frac1(x - 1)^2big) text by L'Hôpital \
&= 2 lim_x to 1^+frac-3(x - 1)^-4-2expbig(frac1(x - 1)^2big)(x - 1)^-3 \
&= 3lim_x to 1^+frac(x - 1)^-1expbig(frac1(x - 1)^2big) quadtextagain, this has the form fracinftyinfty text so L'Hôpital applies \
&= 3 lim_x to 1^+fracfracddx(x - 1)^-1fracddxexpbig(frac1(x - 1)^2big) text by L'Hôpital \
&= 3 lim_x to 1^+frac-(x - 1)^-2-2expbig(frac1(x - 1)^2big)(x - 1)^-3 \
&= frac32 lim_x to 1^+fracx - 1expbig(frac1(x - 1)^2big) \
&= frac32 lim_x to 1^+ Big[(x - 1)expBig(-frac1(x - 1)^2Big)Big] \
&= frac32 Big[lim_x to 1^+ (x - 1)Big] Big[lim_x to 1^+ expBig(-frac1(x - 1)^2Big)Big] = frac32 times 0 times 0 = 0
endalign*
That was a long calculation but take my word: it can be repeated inductively to show that $limlimits_x to 1+fracd^nfdx^n = 0$ for all $n in mathbbZ_+!$ At all other points i.e. on $(-infty, -1) cup (-1, 1) cup (1, infty)$, $f$ is infinitely differentiable because exponentials and constant functions are infinitely differentiable.



Bonus Fact:



Both $fracd^n f(x)dx^n$ and $fracd^n g(x)dx^n$ also have the same value $0$ on $[-1, 1]$ for all positive integers $n$!



Edit:



A more elementary way of seeing $lim_x to 1^+fracdf(x)dx = 0$ is as follows:
beginalign*
lim_x to 1^+fracdf(x)dx &= 2lim_x to 1^+fracfrac1(x - 1)^3expbig(frac1(x - 1)^2big) \
&= 2lim_y to +inftyfracy^3expbig(y^2big) quadtext changing variables to y = frac1x - 1 \
&= 2lim_y to +inftyfracy^3sumlimits_n = 0^infty fracy^2nn! \
&= 2lim_y to +inftyfracy^31 + y^2 + fracy^42 + sumlimits_n = 3^infty fracy^2nn! \
&= 2lim_y to +inftyfrac1frac1y^3 + frac1y + fracy2 + sumlimits_n = 3^infty fracy^2n - 3n! quadtext dividing top and bottom by y^3 \
&= 2 times 0 = 0
endalign*
Essentially, the top term $1$, which remains constant, is a sitting duck. Meanwhile the bottom term blows up to infinity because in the bottom, only two terms ($frac1y^3$ and $frac1y$) go to zero as $y$ becomes large. The rest of the terms ($fracy2$ and $fracy^2n - 3n!$ for $n in mathbbZ_+ geq 3$) have at least a $y$ in the numerator and since no cancellation happens (all terms are positive), their sum becomes arbitrarily large and the overall ratio becomes arbitrarily small.






share|cite|improve this answer











$endgroup$








  • 2




    $begingroup$
    It basically says under certain conditions, $limlimits_x to a(f(x) / g(x)) = limlimits_x to a(fracd f(x)dx / fracd g(x)dx)$. en.wikipedia.org/wiki/L%27H%C3%B4pital%27s_rule
    $endgroup$
    – ZeroXLR
    Apr 12 at 22:49







  • 1




    $begingroup$
    I am stunned. Do you know of more complex solutions?
    $endgroup$
    – TVSuchty
    Apr 12 at 22:52






  • 1




    $begingroup$
    @TVSuchty what class shows you logic symbols before explaining L'Hôpitals rule?
    $endgroup$
    – uhhhhidk
    Apr 13 at 0:45






  • 1




    $begingroup$
    We can obtain $f^(n)(1)=0$ for all $n$ without l'Hopital's Rule by induction and some elementary considerations........+1
    $endgroup$
    – DanielWainfleet
    Apr 13 at 6:41







  • 1




    $begingroup$
    @DanielWainfleet Ah yes, I should probably add an edit with an elementary proof since the OP admitted he did not cover L'Hôpital.
    $endgroup$
    – ZeroXLR
    Apr 13 at 6:59


















5












$begingroup$

Sure. In fact, there is a whole class of functions which not only exist, but are specifically made to do something that effectively implies exactly what you are looking for: they are called bump functions, and are defined as smooth (differentiable everywhere, arbitrarily many times) functions that have compact support, meaning (almost) that they are zero everywhere except on a compact set, which for the real numbers as domain basically means a closed, bounded (i.e. contained within an interval) subset thereof, such as a closed interval. This compact set where they are nonzero is called the "support". The trick is to exploit the "everywhere else zero"-ness, as that gives you what you're after.



Namely, any two different (i.e. not equal) bump functions with the same supporting interval $[a, b]$, will be smooth, zero on any interval outside this interval, and yet different, because they differ on such interval. More generally, given any two different bump functions, period, you just have to find an interval outside both of their support sets, which is always possible because they are both bounded.



A simple example of such a bump function is



$$mathrmbump: mathbbR rightarrow mathbbR, mathrmbump(x) := begincases e^-frac11 - x^2, mboxwhen $x$ is in $(-1, 1)$\ 0, mboxotherwise endcases$$



Then consider just $mathrmbump(x)$ and a nonzero multiple thereof, say, $mathrmbump2(x) := 2 cdot mathrmbump(x)$. We now have $mathrmbump(x) = mathrmbump2(x)$ when, say, $x in [10, 11]$, since they are both zero there. Yet, they are ostensibly not equal when $x$ is in $(-1, 1)$.



ADD: It appears to have been asked as to how one can do this without explicitly constructing the bump function. The above is just to show (not completely thoroughly) that bump functions exist. Indeed, we can do so as well. Let now $mathrmbump(x)$ be a general bump function. Let its support set be $mathrmsupp[mathrmbump]$. That is,



$$mathrmsupp[mathrmbump] := mathrmclleft( x in mathbbR : mathrmbump(x) ne 0 right)$$



(n.b. "cl" means to take the closure; basically this includes all "endpoints" of regions in which it is nonzero, even if it is zero at those endpoints - e.g. the support of the just-given-explicitly bump function is $[-1, 1]$, not $(-1, 1)$. This is a bit of technicality that was wrapped earlier when I said "almost" in "meaning (almost)" above.)



Since the support set is bounded and closed, it has a maximum and minimum (largest and smallest element): assign $M := mathrmmax mathrmsupp[mathrmbump]$. Now consider the interval $mathrmext ival := [M+1, M+2]$. If $x in mathrmext ival$, then it is clearly not in the support set, but rather to the right of it. Thus $mathrmbump(x) = 0$ there. Now set $mathrmbump2(x) := 2 cdot mathrmbump(x)$ as before (if you want even more generality, just replace $2$ with an arbitrary vertical rescaling coefficient $a$ that is not $0$ or $1$). Congrats, you now have two bump functions that are unequal but equal on the external interval $mathrmext ival$.






share|cite|improve this answer











$endgroup$








  • 1




    $begingroup$
    I see what you did there! Is there a case, where you can write an example without defining the function bit by bit, maybe just as term?
    $endgroup$
    – TVSuchty
    Apr 13 at 9:08






  • 2




    $begingroup$
    @TVSuchty : Actually, yes. See what I just added. I believe this is what you're asking about.
    $endgroup$
    – The_Sympathizer
    Apr 13 at 14:21






  • 1




    $begingroup$
    @TVSuchty : "I see what you did there!" yeah I just made the little bumpzzle perk up :) #mehhr.
    $endgroup$
    – The_Sympathizer
    Apr 13 at 14:45










  • $begingroup$
    @TVSuchty piecewise defined functions are not special and you shouldn't see functions "just as term" as not being piecewise either.
    $endgroup$
    – Tony S.F.
    12 hours ago











Your Answer








StackExchange.ready(function()
var channelOptions =
tags: "".split(" "),
id: "69"
;
initTagRenderer("".split(" "), "".split(" "), channelOptions);

StackExchange.using("externalEditor", function()
// Have to fire editor after snippets, if snippets enabled
if (StackExchange.settings.snippets.snippetsEnabled)
StackExchange.using("snippets", function()
createEditor();
);

else
createEditor();

);

function createEditor()
StackExchange.prepareEditor(
heartbeatType: 'answer',
autoActivateHeartbeat: false,
convertImagesToLinks: true,
noModals: true,
showLowRepImageUploadWarning: true,
reputationToPostImages: 10,
bindNavPrevention: true,
postfix: "",
imageUploader:
brandingHtml: "Powered by u003ca class="icon-imgur-white" href="https://imgur.com/"u003eu003c/au003e",
contentPolicyHtml: "User contributions licensed under u003ca href="https://creativecommons.org/licenses/by-sa/3.0/"u003ecc by-sa 3.0 with attribution requiredu003c/au003e u003ca href="https://stackoverflow.com/legal/content-policy"u003e(content policy)u003c/au003e",
allowUrls: true
,
noCode: true, onDemand: true,
discardSelector: ".discard-answer"
,immediatelyShowMarkdownHelp:true
);



);













draft saved

draft discarded


















StackExchange.ready(
function ()
StackExchange.openid.initPostLogin('.new-post-login', 'https%3a%2f%2fmath.stackexchange.com%2fquestions%2f3185630%2fare-there-continuous-functions-who-are-the-same-in-an-interval-but-differ-in-at%23new-answer', 'question_page');

);

Post as a guest















Required, but never shown

























2 Answers
2






active

oldest

votes








2 Answers
2






active

oldest

votes









active

oldest

votes






active

oldest

votes









18












$begingroup$

Define the real functions $f$ and $g$ thus:
$$
f(x) = begincases expBig(-frac1(x - 1)^2Big) &textif x > 1 \
0 &textif x in [-1, 1] \
expBig(-frac1(x + 1)^2Big) &textif x < -1
endcases
$$
and
$g(x) = 0$. $f$ and $g$ are both $0$ on $[-1, 1]$ but they differ in value everywhere else.



Obviously $g$ is continuously differentiable infinitely many times as it is a constant function. You can also check that $f$ is continuously differentiable infinitely many times at $x = -1$ and $x = 1$ by applying L'Hôpital's rule inductively (Edit: A more elementary demonstration is given below based on the exponential series). Checking this is a fine exercise in Real Analysis; you should try it. Here is a first taste of it:
beginalign*
lim_x to 1^+fracdf(x)dx &= limlimits_x to 1^+frac2expbig(- frac1(x - 1)^2big)(x - 1)^3 \
&= 2lim_x to 1^+fracfrac1(x - 1)^3expbig(frac1(x - 1)^2big) quadtextthis limit is of the form fracinftyinfty text so L'Hôpital applies \
&= 2 lim_x to 1^+fracfracddx(x - 1)^-3fracddxexpbig(frac1(x - 1)^2big) text by L'Hôpital \
&= 2 lim_x to 1^+frac-3(x - 1)^-4-2expbig(frac1(x - 1)^2big)(x - 1)^-3 \
&= 3lim_x to 1^+frac(x - 1)^-1expbig(frac1(x - 1)^2big) quadtextagain, this has the form fracinftyinfty text so L'Hôpital applies \
&= 3 lim_x to 1^+fracfracddx(x - 1)^-1fracddxexpbig(frac1(x - 1)^2big) text by L'Hôpital \
&= 3 lim_x to 1^+frac-(x - 1)^-2-2expbig(frac1(x - 1)^2big)(x - 1)^-3 \
&= frac32 lim_x to 1^+fracx - 1expbig(frac1(x - 1)^2big) \
&= frac32 lim_x to 1^+ Big[(x - 1)expBig(-frac1(x - 1)^2Big)Big] \
&= frac32 Big[lim_x to 1^+ (x - 1)Big] Big[lim_x to 1^+ expBig(-frac1(x - 1)^2Big)Big] = frac32 times 0 times 0 = 0
endalign*
That was a long calculation but take my word: it can be repeated inductively to show that $limlimits_x to 1+fracd^nfdx^n = 0$ for all $n in mathbbZ_+!$ At all other points i.e. on $(-infty, -1) cup (-1, 1) cup (1, infty)$, $f$ is infinitely differentiable because exponentials and constant functions are infinitely differentiable.



Bonus Fact:



Both $fracd^n f(x)dx^n$ and $fracd^n g(x)dx^n$ also have the same value $0$ on $[-1, 1]$ for all positive integers $n$!



Edit:



A more elementary way of seeing $lim_x to 1^+fracdf(x)dx = 0$ is as follows:
beginalign*
lim_x to 1^+fracdf(x)dx &= 2lim_x to 1^+fracfrac1(x - 1)^3expbig(frac1(x - 1)^2big) \
&= 2lim_y to +inftyfracy^3expbig(y^2big) quadtext changing variables to y = frac1x - 1 \
&= 2lim_y to +inftyfracy^3sumlimits_n = 0^infty fracy^2nn! \
&= 2lim_y to +inftyfracy^31 + y^2 + fracy^42 + sumlimits_n = 3^infty fracy^2nn! \
&= 2lim_y to +inftyfrac1frac1y^3 + frac1y + fracy2 + sumlimits_n = 3^infty fracy^2n - 3n! quadtext dividing top and bottom by y^3 \
&= 2 times 0 = 0
endalign*
Essentially, the top term $1$, which remains constant, is a sitting duck. Meanwhile the bottom term blows up to infinity because in the bottom, only two terms ($frac1y^3$ and $frac1y$) go to zero as $y$ becomes large. The rest of the terms ($fracy2$ and $fracy^2n - 3n!$ for $n in mathbbZ_+ geq 3$) have at least a $y$ in the numerator and since no cancellation happens (all terms are positive), their sum becomes arbitrarily large and the overall ratio becomes arbitrarily small.






share|cite|improve this answer











$endgroup$








  • 2




    $begingroup$
    It basically says under certain conditions, $limlimits_x to a(f(x) / g(x)) = limlimits_x to a(fracd f(x)dx / fracd g(x)dx)$. en.wikipedia.org/wiki/L%27H%C3%B4pital%27s_rule
    $endgroup$
    – ZeroXLR
    Apr 12 at 22:49







  • 1




    $begingroup$
    I am stunned. Do you know of more complex solutions?
    $endgroup$
    – TVSuchty
    Apr 12 at 22:52






  • 1




    $begingroup$
    @TVSuchty what class shows you logic symbols before explaining L'Hôpitals rule?
    $endgroup$
    – uhhhhidk
    Apr 13 at 0:45






  • 1




    $begingroup$
    We can obtain $f^(n)(1)=0$ for all $n$ without l'Hopital's Rule by induction and some elementary considerations........+1
    $endgroup$
    – DanielWainfleet
    Apr 13 at 6:41







  • 1




    $begingroup$
    @DanielWainfleet Ah yes, I should probably add an edit with an elementary proof since the OP admitted he did not cover L'Hôpital.
    $endgroup$
    – ZeroXLR
    Apr 13 at 6:59















18












$begingroup$

Define the real functions $f$ and $g$ thus:
$$
f(x) = begincases expBig(-frac1(x - 1)^2Big) &textif x > 1 \
0 &textif x in [-1, 1] \
expBig(-frac1(x + 1)^2Big) &textif x < -1
endcases
$$
and
$g(x) = 0$. $f$ and $g$ are both $0$ on $[-1, 1]$ but they differ in value everywhere else.



Obviously $g$ is continuously differentiable infinitely many times as it is a constant function. You can also check that $f$ is continuously differentiable infinitely many times at $x = -1$ and $x = 1$ by applying L'Hôpital's rule inductively (Edit: A more elementary demonstration is given below based on the exponential series). Checking this is a fine exercise in Real Analysis; you should try it. Here is a first taste of it:
beginalign*
lim_x to 1^+fracdf(x)dx &= limlimits_x to 1^+frac2expbig(- frac1(x - 1)^2big)(x - 1)^3 \
&= 2lim_x to 1^+fracfrac1(x - 1)^3expbig(frac1(x - 1)^2big) quadtextthis limit is of the form fracinftyinfty text so L'Hôpital applies \
&= 2 lim_x to 1^+fracfracddx(x - 1)^-3fracddxexpbig(frac1(x - 1)^2big) text by L'Hôpital \
&= 2 lim_x to 1^+frac-3(x - 1)^-4-2expbig(frac1(x - 1)^2big)(x - 1)^-3 \
&= 3lim_x to 1^+frac(x - 1)^-1expbig(frac1(x - 1)^2big) quadtextagain, this has the form fracinftyinfty text so L'Hôpital applies \
&= 3 lim_x to 1^+fracfracddx(x - 1)^-1fracddxexpbig(frac1(x - 1)^2big) text by L'Hôpital \
&= 3 lim_x to 1^+frac-(x - 1)^-2-2expbig(frac1(x - 1)^2big)(x - 1)^-3 \
&= frac32 lim_x to 1^+fracx - 1expbig(frac1(x - 1)^2big) \
&= frac32 lim_x to 1^+ Big[(x - 1)expBig(-frac1(x - 1)^2Big)Big] \
&= frac32 Big[lim_x to 1^+ (x - 1)Big] Big[lim_x to 1^+ expBig(-frac1(x - 1)^2Big)Big] = frac32 times 0 times 0 = 0
endalign*
That was a long calculation but take my word: it can be repeated inductively to show that $limlimits_x to 1+fracd^nfdx^n = 0$ for all $n in mathbbZ_+!$ At all other points i.e. on $(-infty, -1) cup (-1, 1) cup (1, infty)$, $f$ is infinitely differentiable because exponentials and constant functions are infinitely differentiable.



Bonus Fact:



Both $fracd^n f(x)dx^n$ and $fracd^n g(x)dx^n$ also have the same value $0$ on $[-1, 1]$ for all positive integers $n$!



Edit:



A more elementary way of seeing $lim_x to 1^+fracdf(x)dx = 0$ is as follows:
beginalign*
lim_x to 1^+fracdf(x)dx &= 2lim_x to 1^+fracfrac1(x - 1)^3expbig(frac1(x - 1)^2big) \
&= 2lim_y to +inftyfracy^3expbig(y^2big) quadtext changing variables to y = frac1x - 1 \
&= 2lim_y to +inftyfracy^3sumlimits_n = 0^infty fracy^2nn! \
&= 2lim_y to +inftyfracy^31 + y^2 + fracy^42 + sumlimits_n = 3^infty fracy^2nn! \
&= 2lim_y to +inftyfrac1frac1y^3 + frac1y + fracy2 + sumlimits_n = 3^infty fracy^2n - 3n! quadtext dividing top and bottom by y^3 \
&= 2 times 0 = 0
endalign*
Essentially, the top term $1$, which remains constant, is a sitting duck. Meanwhile the bottom term blows up to infinity because in the bottom, only two terms ($frac1y^3$ and $frac1y$) go to zero as $y$ becomes large. The rest of the terms ($fracy2$ and $fracy^2n - 3n!$ for $n in mathbbZ_+ geq 3$) have at least a $y$ in the numerator and since no cancellation happens (all terms are positive), their sum becomes arbitrarily large and the overall ratio becomes arbitrarily small.






share|cite|improve this answer











$endgroup$








  • 2




    $begingroup$
    It basically says under certain conditions, $limlimits_x to a(f(x) / g(x)) = limlimits_x to a(fracd f(x)dx / fracd g(x)dx)$. en.wikipedia.org/wiki/L%27H%C3%B4pital%27s_rule
    $endgroup$
    – ZeroXLR
    Apr 12 at 22:49







  • 1




    $begingroup$
    I am stunned. Do you know of more complex solutions?
    $endgroup$
    – TVSuchty
    Apr 12 at 22:52






  • 1




    $begingroup$
    @TVSuchty what class shows you logic symbols before explaining L'Hôpitals rule?
    $endgroup$
    – uhhhhidk
    Apr 13 at 0:45






  • 1




    $begingroup$
    We can obtain $f^(n)(1)=0$ for all $n$ without l'Hopital's Rule by induction and some elementary considerations........+1
    $endgroup$
    – DanielWainfleet
    Apr 13 at 6:41







  • 1




    $begingroup$
    @DanielWainfleet Ah yes, I should probably add an edit with an elementary proof since the OP admitted he did not cover L'Hôpital.
    $endgroup$
    – ZeroXLR
    Apr 13 at 6:59













18












18








18





$begingroup$

Define the real functions $f$ and $g$ thus:
$$
f(x) = begincases expBig(-frac1(x - 1)^2Big) &textif x > 1 \
0 &textif x in [-1, 1] \
expBig(-frac1(x + 1)^2Big) &textif x < -1
endcases
$$
and
$g(x) = 0$. $f$ and $g$ are both $0$ on $[-1, 1]$ but they differ in value everywhere else.



Obviously $g$ is continuously differentiable infinitely many times as it is a constant function. You can also check that $f$ is continuously differentiable infinitely many times at $x = -1$ and $x = 1$ by applying L'Hôpital's rule inductively (Edit: A more elementary demonstration is given below based on the exponential series). Checking this is a fine exercise in Real Analysis; you should try it. Here is a first taste of it:
beginalign*
lim_x to 1^+fracdf(x)dx &= limlimits_x to 1^+frac2expbig(- frac1(x - 1)^2big)(x - 1)^3 \
&= 2lim_x to 1^+fracfrac1(x - 1)^3expbig(frac1(x - 1)^2big) quadtextthis limit is of the form fracinftyinfty text so L'Hôpital applies \
&= 2 lim_x to 1^+fracfracddx(x - 1)^-3fracddxexpbig(frac1(x - 1)^2big) text by L'Hôpital \
&= 2 lim_x to 1^+frac-3(x - 1)^-4-2expbig(frac1(x - 1)^2big)(x - 1)^-3 \
&= 3lim_x to 1^+frac(x - 1)^-1expbig(frac1(x - 1)^2big) quadtextagain, this has the form fracinftyinfty text so L'Hôpital applies \
&= 3 lim_x to 1^+fracfracddx(x - 1)^-1fracddxexpbig(frac1(x - 1)^2big) text by L'Hôpital \
&= 3 lim_x to 1^+frac-(x - 1)^-2-2expbig(frac1(x - 1)^2big)(x - 1)^-3 \
&= frac32 lim_x to 1^+fracx - 1expbig(frac1(x - 1)^2big) \
&= frac32 lim_x to 1^+ Big[(x - 1)expBig(-frac1(x - 1)^2Big)Big] \
&= frac32 Big[lim_x to 1^+ (x - 1)Big] Big[lim_x to 1^+ expBig(-frac1(x - 1)^2Big)Big] = frac32 times 0 times 0 = 0
endalign*
That was a long calculation but take my word: it can be repeated inductively to show that $limlimits_x to 1+fracd^nfdx^n = 0$ for all $n in mathbbZ_+!$ At all other points i.e. on $(-infty, -1) cup (-1, 1) cup (1, infty)$, $f$ is infinitely differentiable because exponentials and constant functions are infinitely differentiable.



Bonus Fact:



Both $fracd^n f(x)dx^n$ and $fracd^n g(x)dx^n$ also have the same value $0$ on $[-1, 1]$ for all positive integers $n$!



Edit:



A more elementary way of seeing $lim_x to 1^+fracdf(x)dx = 0$ is as follows:
beginalign*
lim_x to 1^+fracdf(x)dx &= 2lim_x to 1^+fracfrac1(x - 1)^3expbig(frac1(x - 1)^2big) \
&= 2lim_y to +inftyfracy^3expbig(y^2big) quadtext changing variables to y = frac1x - 1 \
&= 2lim_y to +inftyfracy^3sumlimits_n = 0^infty fracy^2nn! \
&= 2lim_y to +inftyfracy^31 + y^2 + fracy^42 + sumlimits_n = 3^infty fracy^2nn! \
&= 2lim_y to +inftyfrac1frac1y^3 + frac1y + fracy2 + sumlimits_n = 3^infty fracy^2n - 3n! quadtext dividing top and bottom by y^3 \
&= 2 times 0 = 0
endalign*
Essentially, the top term $1$, which remains constant, is a sitting duck. Meanwhile the bottom term blows up to infinity because in the bottom, only two terms ($frac1y^3$ and $frac1y$) go to zero as $y$ becomes large. The rest of the terms ($fracy2$ and $fracy^2n - 3n!$ for $n in mathbbZ_+ geq 3$) have at least a $y$ in the numerator and since no cancellation happens (all terms are positive), their sum becomes arbitrarily large and the overall ratio becomes arbitrarily small.






share|cite|improve this answer











$endgroup$



Define the real functions $f$ and $g$ thus:
$$
f(x) = begincases expBig(-frac1(x - 1)^2Big) &textif x > 1 \
0 &textif x in [-1, 1] \
expBig(-frac1(x + 1)^2Big) &textif x < -1
endcases
$$
and
$g(x) = 0$. $f$ and $g$ are both $0$ on $[-1, 1]$ but they differ in value everywhere else.



Obviously $g$ is continuously differentiable infinitely many times as it is a constant function. You can also check that $f$ is continuously differentiable infinitely many times at $x = -1$ and $x = 1$ by applying L'Hôpital's rule inductively (Edit: A more elementary demonstration is given below based on the exponential series). Checking this is a fine exercise in Real Analysis; you should try it. Here is a first taste of it:
beginalign*
lim_x to 1^+fracdf(x)dx &= limlimits_x to 1^+frac2expbig(- frac1(x - 1)^2big)(x - 1)^3 \
&= 2lim_x to 1^+fracfrac1(x - 1)^3expbig(frac1(x - 1)^2big) quadtextthis limit is of the form fracinftyinfty text so L'Hôpital applies \
&= 2 lim_x to 1^+fracfracddx(x - 1)^-3fracddxexpbig(frac1(x - 1)^2big) text by L'Hôpital \
&= 2 lim_x to 1^+frac-3(x - 1)^-4-2expbig(frac1(x - 1)^2big)(x - 1)^-3 \
&= 3lim_x to 1^+frac(x - 1)^-1expbig(frac1(x - 1)^2big) quadtextagain, this has the form fracinftyinfty text so L'Hôpital applies \
&= 3 lim_x to 1^+fracfracddx(x - 1)^-1fracddxexpbig(frac1(x - 1)^2big) text by L'Hôpital \
&= 3 lim_x to 1^+frac-(x - 1)^-2-2expbig(frac1(x - 1)^2big)(x - 1)^-3 \
&= frac32 lim_x to 1^+fracx - 1expbig(frac1(x - 1)^2big) \
&= frac32 lim_x to 1^+ Big[(x - 1)expBig(-frac1(x - 1)^2Big)Big] \
&= frac32 Big[lim_x to 1^+ (x - 1)Big] Big[lim_x to 1^+ expBig(-frac1(x - 1)^2Big)Big] = frac32 times 0 times 0 = 0
endalign*
That was a long calculation but take my word: it can be repeated inductively to show that $limlimits_x to 1+fracd^nfdx^n = 0$ for all $n in mathbbZ_+!$ At all other points i.e. on $(-infty, -1) cup (-1, 1) cup (1, infty)$, $f$ is infinitely differentiable because exponentials and constant functions are infinitely differentiable.



Bonus Fact:



Both $fracd^n f(x)dx^n$ and $fracd^n g(x)dx^n$ also have the same value $0$ on $[-1, 1]$ for all positive integers $n$!



Edit:



A more elementary way of seeing $lim_x to 1^+fracdf(x)dx = 0$ is as follows:
beginalign*
lim_x to 1^+fracdf(x)dx &= 2lim_x to 1^+fracfrac1(x - 1)^3expbig(frac1(x - 1)^2big) \
&= 2lim_y to +inftyfracy^3expbig(y^2big) quadtext changing variables to y = frac1x - 1 \
&= 2lim_y to +inftyfracy^3sumlimits_n = 0^infty fracy^2nn! \
&= 2lim_y to +inftyfracy^31 + y^2 + fracy^42 + sumlimits_n = 3^infty fracy^2nn! \
&= 2lim_y to +inftyfrac1frac1y^3 + frac1y + fracy2 + sumlimits_n = 3^infty fracy^2n - 3n! quadtext dividing top and bottom by y^3 \
&= 2 times 0 = 0
endalign*
Essentially, the top term $1$, which remains constant, is a sitting duck. Meanwhile the bottom term blows up to infinity because in the bottom, only two terms ($frac1y^3$ and $frac1y$) go to zero as $y$ becomes large. The rest of the terms ($fracy2$ and $fracy^2n - 3n!$ for $n in mathbbZ_+ geq 3$) have at least a $y$ in the numerator and since no cancellation happens (all terms are positive), their sum becomes arbitrarily large and the overall ratio becomes arbitrarily small.







share|cite|improve this answer














share|cite|improve this answer



share|cite|improve this answer








edited Apr 13 at 7:22

























answered Apr 12 at 22:39









ZeroXLRZeroXLR

1,780620




1,780620







  • 2




    $begingroup$
    It basically says under certain conditions, $limlimits_x to a(f(x) / g(x)) = limlimits_x to a(fracd f(x)dx / fracd g(x)dx)$. en.wikipedia.org/wiki/L%27H%C3%B4pital%27s_rule
    $endgroup$
    – ZeroXLR
    Apr 12 at 22:49







  • 1




    $begingroup$
    I am stunned. Do you know of more complex solutions?
    $endgroup$
    – TVSuchty
    Apr 12 at 22:52






  • 1




    $begingroup$
    @TVSuchty what class shows you logic symbols before explaining L'Hôpitals rule?
    $endgroup$
    – uhhhhidk
    Apr 13 at 0:45






  • 1




    $begingroup$
    We can obtain $f^(n)(1)=0$ for all $n$ without l'Hopital's Rule by induction and some elementary considerations........+1
    $endgroup$
    – DanielWainfleet
    Apr 13 at 6:41







  • 1




    $begingroup$
    @DanielWainfleet Ah yes, I should probably add an edit with an elementary proof since the OP admitted he did not cover L'Hôpital.
    $endgroup$
    – ZeroXLR
    Apr 13 at 6:59












  • 2




    $begingroup$
    It basically says under certain conditions, $limlimits_x to a(f(x) / g(x)) = limlimits_x to a(fracd f(x)dx / fracd g(x)dx)$. en.wikipedia.org/wiki/L%27H%C3%B4pital%27s_rule
    $endgroup$
    – ZeroXLR
    Apr 12 at 22:49







  • 1




    $begingroup$
    I am stunned. Do you know of more complex solutions?
    $endgroup$
    – TVSuchty
    Apr 12 at 22:52






  • 1




    $begingroup$
    @TVSuchty what class shows you logic symbols before explaining L'Hôpitals rule?
    $endgroup$
    – uhhhhidk
    Apr 13 at 0:45






  • 1




    $begingroup$
    We can obtain $f^(n)(1)=0$ for all $n$ without l'Hopital's Rule by induction and some elementary considerations........+1
    $endgroup$
    – DanielWainfleet
    Apr 13 at 6:41







  • 1




    $begingroup$
    @DanielWainfleet Ah yes, I should probably add an edit with an elementary proof since the OP admitted he did not cover L'Hôpital.
    $endgroup$
    – ZeroXLR
    Apr 13 at 6:59







2




2




$begingroup$
It basically says under certain conditions, $limlimits_x to a(f(x) / g(x)) = limlimits_x to a(fracd f(x)dx / fracd g(x)dx)$. en.wikipedia.org/wiki/L%27H%C3%B4pital%27s_rule
$endgroup$
– ZeroXLR
Apr 12 at 22:49





$begingroup$
It basically says under certain conditions, $limlimits_x to a(f(x) / g(x)) = limlimits_x to a(fracd f(x)dx / fracd g(x)dx)$. en.wikipedia.org/wiki/L%27H%C3%B4pital%27s_rule
$endgroup$
– ZeroXLR
Apr 12 at 22:49





1




1




$begingroup$
I am stunned. Do you know of more complex solutions?
$endgroup$
– TVSuchty
Apr 12 at 22:52




$begingroup$
I am stunned. Do you know of more complex solutions?
$endgroup$
– TVSuchty
Apr 12 at 22:52




1




1




$begingroup$
@TVSuchty what class shows you logic symbols before explaining L'Hôpitals rule?
$endgroup$
– uhhhhidk
Apr 13 at 0:45




$begingroup$
@TVSuchty what class shows you logic symbols before explaining L'Hôpitals rule?
$endgroup$
– uhhhhidk
Apr 13 at 0:45




1




1




$begingroup$
We can obtain $f^(n)(1)=0$ for all $n$ without l'Hopital's Rule by induction and some elementary considerations........+1
$endgroup$
– DanielWainfleet
Apr 13 at 6:41





$begingroup$
We can obtain $f^(n)(1)=0$ for all $n$ without l'Hopital's Rule by induction and some elementary considerations........+1
$endgroup$
– DanielWainfleet
Apr 13 at 6:41





1




1




$begingroup$
@DanielWainfleet Ah yes, I should probably add an edit with an elementary proof since the OP admitted he did not cover L'Hôpital.
$endgroup$
– ZeroXLR
Apr 13 at 6:59




$begingroup$
@DanielWainfleet Ah yes, I should probably add an edit with an elementary proof since the OP admitted he did not cover L'Hôpital.
$endgroup$
– ZeroXLR
Apr 13 at 6:59











5












$begingroup$

Sure. In fact, there is a whole class of functions which not only exist, but are specifically made to do something that effectively implies exactly what you are looking for: they are called bump functions, and are defined as smooth (differentiable everywhere, arbitrarily many times) functions that have compact support, meaning (almost) that they are zero everywhere except on a compact set, which for the real numbers as domain basically means a closed, bounded (i.e. contained within an interval) subset thereof, such as a closed interval. This compact set where they are nonzero is called the "support". The trick is to exploit the "everywhere else zero"-ness, as that gives you what you're after.



Namely, any two different (i.e. not equal) bump functions with the same supporting interval $[a, b]$, will be smooth, zero on any interval outside this interval, and yet different, because they differ on such interval. More generally, given any two different bump functions, period, you just have to find an interval outside both of their support sets, which is always possible because they are both bounded.



A simple example of such a bump function is



$$mathrmbump: mathbbR rightarrow mathbbR, mathrmbump(x) := begincases e^-frac11 - x^2, mboxwhen $x$ is in $(-1, 1)$\ 0, mboxotherwise endcases$$



Then consider just $mathrmbump(x)$ and a nonzero multiple thereof, say, $mathrmbump2(x) := 2 cdot mathrmbump(x)$. We now have $mathrmbump(x) = mathrmbump2(x)$ when, say, $x in [10, 11]$, since they are both zero there. Yet, they are ostensibly not equal when $x$ is in $(-1, 1)$.



ADD: It appears to have been asked as to how one can do this without explicitly constructing the bump function. The above is just to show (not completely thoroughly) that bump functions exist. Indeed, we can do so as well. Let now $mathrmbump(x)$ be a general bump function. Let its support set be $mathrmsupp[mathrmbump]$. That is,



$$mathrmsupp[mathrmbump] := mathrmclleft( x in mathbbR : mathrmbump(x) ne 0 right)$$



(n.b. "cl" means to take the closure; basically this includes all "endpoints" of regions in which it is nonzero, even if it is zero at those endpoints - e.g. the support of the just-given-explicitly bump function is $[-1, 1]$, not $(-1, 1)$. This is a bit of technicality that was wrapped earlier when I said "almost" in "meaning (almost)" above.)



Since the support set is bounded and closed, it has a maximum and minimum (largest and smallest element): assign $M := mathrmmax mathrmsupp[mathrmbump]$. Now consider the interval $mathrmext ival := [M+1, M+2]$. If $x in mathrmext ival$, then it is clearly not in the support set, but rather to the right of it. Thus $mathrmbump(x) = 0$ there. Now set $mathrmbump2(x) := 2 cdot mathrmbump(x)$ as before (if you want even more generality, just replace $2$ with an arbitrary vertical rescaling coefficient $a$ that is not $0$ or $1$). Congrats, you now have two bump functions that are unequal but equal on the external interval $mathrmext ival$.






share|cite|improve this answer











$endgroup$








  • 1




    $begingroup$
    I see what you did there! Is there a case, where you can write an example without defining the function bit by bit, maybe just as term?
    $endgroup$
    – TVSuchty
    Apr 13 at 9:08






  • 2




    $begingroup$
    @TVSuchty : Actually, yes. See what I just added. I believe this is what you're asking about.
    $endgroup$
    – The_Sympathizer
    Apr 13 at 14:21






  • 1




    $begingroup$
    @TVSuchty : "I see what you did there!" yeah I just made the little bumpzzle perk up :) #mehhr.
    $endgroup$
    – The_Sympathizer
    Apr 13 at 14:45










  • $begingroup$
    @TVSuchty piecewise defined functions are not special and you shouldn't see functions "just as term" as not being piecewise either.
    $endgroup$
    – Tony S.F.
    12 hours ago















5












$begingroup$

Sure. In fact, there is a whole class of functions which not only exist, but are specifically made to do something that effectively implies exactly what you are looking for: they are called bump functions, and are defined as smooth (differentiable everywhere, arbitrarily many times) functions that have compact support, meaning (almost) that they are zero everywhere except on a compact set, which for the real numbers as domain basically means a closed, bounded (i.e. contained within an interval) subset thereof, such as a closed interval. This compact set where they are nonzero is called the "support". The trick is to exploit the "everywhere else zero"-ness, as that gives you what you're after.



Namely, any two different (i.e. not equal) bump functions with the same supporting interval $[a, b]$, will be smooth, zero on any interval outside this interval, and yet different, because they differ on such interval. More generally, given any two different bump functions, period, you just have to find an interval outside both of their support sets, which is always possible because they are both bounded.



A simple example of such a bump function is



$$mathrmbump: mathbbR rightarrow mathbbR, mathrmbump(x) := begincases e^-frac11 - x^2, mboxwhen $x$ is in $(-1, 1)$\ 0, mboxotherwise endcases$$



Then consider just $mathrmbump(x)$ and a nonzero multiple thereof, say, $mathrmbump2(x) := 2 cdot mathrmbump(x)$. We now have $mathrmbump(x) = mathrmbump2(x)$ when, say, $x in [10, 11]$, since they are both zero there. Yet, they are ostensibly not equal when $x$ is in $(-1, 1)$.



ADD: It appears to have been asked as to how one can do this without explicitly constructing the bump function. The above is just to show (not completely thoroughly) that bump functions exist. Indeed, we can do so as well. Let now $mathrmbump(x)$ be a general bump function. Let its support set be $mathrmsupp[mathrmbump]$. That is,



$$mathrmsupp[mathrmbump] := mathrmclleft( x in mathbbR : mathrmbump(x) ne 0 right)$$



(n.b. "cl" means to take the closure; basically this includes all "endpoints" of regions in which it is nonzero, even if it is zero at those endpoints - e.g. the support of the just-given-explicitly bump function is $[-1, 1]$, not $(-1, 1)$. This is a bit of technicality that was wrapped earlier when I said "almost" in "meaning (almost)" above.)



Since the support set is bounded and closed, it has a maximum and minimum (largest and smallest element): assign $M := mathrmmax mathrmsupp[mathrmbump]$. Now consider the interval $mathrmext ival := [M+1, M+2]$. If $x in mathrmext ival$, then it is clearly not in the support set, but rather to the right of it. Thus $mathrmbump(x) = 0$ there. Now set $mathrmbump2(x) := 2 cdot mathrmbump(x)$ as before (if you want even more generality, just replace $2$ with an arbitrary vertical rescaling coefficient $a$ that is not $0$ or $1$). Congrats, you now have two bump functions that are unequal but equal on the external interval $mathrmext ival$.






share|cite|improve this answer











$endgroup$








  • 1




    $begingroup$
    I see what you did there! Is there a case, where you can write an example without defining the function bit by bit, maybe just as term?
    $endgroup$
    – TVSuchty
    Apr 13 at 9:08






  • 2




    $begingroup$
    @TVSuchty : Actually, yes. See what I just added. I believe this is what you're asking about.
    $endgroup$
    – The_Sympathizer
    Apr 13 at 14:21






  • 1




    $begingroup$
    @TVSuchty : "I see what you did there!" yeah I just made the little bumpzzle perk up :) #mehhr.
    $endgroup$
    – The_Sympathizer
    Apr 13 at 14:45










  • $begingroup$
    @TVSuchty piecewise defined functions are not special and you shouldn't see functions "just as term" as not being piecewise either.
    $endgroup$
    – Tony S.F.
    12 hours ago













5












5








5





$begingroup$

Sure. In fact, there is a whole class of functions which not only exist, but are specifically made to do something that effectively implies exactly what you are looking for: they are called bump functions, and are defined as smooth (differentiable everywhere, arbitrarily many times) functions that have compact support, meaning (almost) that they are zero everywhere except on a compact set, which for the real numbers as domain basically means a closed, bounded (i.e. contained within an interval) subset thereof, such as a closed interval. This compact set where they are nonzero is called the "support". The trick is to exploit the "everywhere else zero"-ness, as that gives you what you're after.



Namely, any two different (i.e. not equal) bump functions with the same supporting interval $[a, b]$, will be smooth, zero on any interval outside this interval, and yet different, because they differ on such interval. More generally, given any two different bump functions, period, you just have to find an interval outside both of their support sets, which is always possible because they are both bounded.



A simple example of such a bump function is



$$mathrmbump: mathbbR rightarrow mathbbR, mathrmbump(x) := begincases e^-frac11 - x^2, mboxwhen $x$ is in $(-1, 1)$\ 0, mboxotherwise endcases$$



Then consider just $mathrmbump(x)$ and a nonzero multiple thereof, say, $mathrmbump2(x) := 2 cdot mathrmbump(x)$. We now have $mathrmbump(x) = mathrmbump2(x)$ when, say, $x in [10, 11]$, since they are both zero there. Yet, they are ostensibly not equal when $x$ is in $(-1, 1)$.



ADD: It appears to have been asked as to how one can do this without explicitly constructing the bump function. The above is just to show (not completely thoroughly) that bump functions exist. Indeed, we can do so as well. Let now $mathrmbump(x)$ be a general bump function. Let its support set be $mathrmsupp[mathrmbump]$. That is,



$$mathrmsupp[mathrmbump] := mathrmclleft( x in mathbbR : mathrmbump(x) ne 0 right)$$



(n.b. "cl" means to take the closure; basically this includes all "endpoints" of regions in which it is nonzero, even if it is zero at those endpoints - e.g. the support of the just-given-explicitly bump function is $[-1, 1]$, not $(-1, 1)$. This is a bit of technicality that was wrapped earlier when I said "almost" in "meaning (almost)" above.)



Since the support set is bounded and closed, it has a maximum and minimum (largest and smallest element): assign $M := mathrmmax mathrmsupp[mathrmbump]$. Now consider the interval $mathrmext ival := [M+1, M+2]$. If $x in mathrmext ival$, then it is clearly not in the support set, but rather to the right of it. Thus $mathrmbump(x) = 0$ there. Now set $mathrmbump2(x) := 2 cdot mathrmbump(x)$ as before (if you want even more generality, just replace $2$ with an arbitrary vertical rescaling coefficient $a$ that is not $0$ or $1$). Congrats, you now have two bump functions that are unequal but equal on the external interval $mathrmext ival$.






share|cite|improve this answer











$endgroup$



Sure. In fact, there is a whole class of functions which not only exist, but are specifically made to do something that effectively implies exactly what you are looking for: they are called bump functions, and are defined as smooth (differentiable everywhere, arbitrarily many times) functions that have compact support, meaning (almost) that they are zero everywhere except on a compact set, which for the real numbers as domain basically means a closed, bounded (i.e. contained within an interval) subset thereof, such as a closed interval. This compact set where they are nonzero is called the "support". The trick is to exploit the "everywhere else zero"-ness, as that gives you what you're after.



Namely, any two different (i.e. not equal) bump functions with the same supporting interval $[a, b]$, will be smooth, zero on any interval outside this interval, and yet different, because they differ on such interval. More generally, given any two different bump functions, period, you just have to find an interval outside both of their support sets, which is always possible because they are both bounded.



A simple example of such a bump function is



$$mathrmbump: mathbbR rightarrow mathbbR, mathrmbump(x) := begincases e^-frac11 - x^2, mboxwhen $x$ is in $(-1, 1)$\ 0, mboxotherwise endcases$$



Then consider just $mathrmbump(x)$ and a nonzero multiple thereof, say, $mathrmbump2(x) := 2 cdot mathrmbump(x)$. We now have $mathrmbump(x) = mathrmbump2(x)$ when, say, $x in [10, 11]$, since they are both zero there. Yet, they are ostensibly not equal when $x$ is in $(-1, 1)$.



ADD: It appears to have been asked as to how one can do this without explicitly constructing the bump function. The above is just to show (not completely thoroughly) that bump functions exist. Indeed, we can do so as well. Let now $mathrmbump(x)$ be a general bump function. Let its support set be $mathrmsupp[mathrmbump]$. That is,



$$mathrmsupp[mathrmbump] := mathrmclleft( x in mathbbR : mathrmbump(x) ne 0 right)$$



(n.b. "cl" means to take the closure; basically this includes all "endpoints" of regions in which it is nonzero, even if it is zero at those endpoints - e.g. the support of the just-given-explicitly bump function is $[-1, 1]$, not $(-1, 1)$. This is a bit of technicality that was wrapped earlier when I said "almost" in "meaning (almost)" above.)



Since the support set is bounded and closed, it has a maximum and minimum (largest and smallest element): assign $M := mathrmmax mathrmsupp[mathrmbump]$. Now consider the interval $mathrmext ival := [M+1, M+2]$. If $x in mathrmext ival$, then it is clearly not in the support set, but rather to the right of it. Thus $mathrmbump(x) = 0$ there. Now set $mathrmbump2(x) := 2 cdot mathrmbump(x)$ as before (if you want even more generality, just replace $2$ with an arbitrary vertical rescaling coefficient $a$ that is not $0$ or $1$). Congrats, you now have two bump functions that are unequal but equal on the external interval $mathrmext ival$.







share|cite|improve this answer














share|cite|improve this answer



share|cite|improve this answer








edited Apr 13 at 14:27

























answered Apr 13 at 6:43









The_SympathizerThe_Sympathizer

7,9302246




7,9302246







  • 1




    $begingroup$
    I see what you did there! Is there a case, where you can write an example without defining the function bit by bit, maybe just as term?
    $endgroup$
    – TVSuchty
    Apr 13 at 9:08






  • 2




    $begingroup$
    @TVSuchty : Actually, yes. See what I just added. I believe this is what you're asking about.
    $endgroup$
    – The_Sympathizer
    Apr 13 at 14:21






  • 1




    $begingroup$
    @TVSuchty : "I see what you did there!" yeah I just made the little bumpzzle perk up :) #mehhr.
    $endgroup$
    – The_Sympathizer
    Apr 13 at 14:45










  • $begingroup$
    @TVSuchty piecewise defined functions are not special and you shouldn't see functions "just as term" as not being piecewise either.
    $endgroup$
    – Tony S.F.
    12 hours ago












  • 1




    $begingroup$
    I see what you did there! Is there a case, where you can write an example without defining the function bit by bit, maybe just as term?
    $endgroup$
    – TVSuchty
    Apr 13 at 9:08






  • 2




    $begingroup$
    @TVSuchty : Actually, yes. See what I just added. I believe this is what you're asking about.
    $endgroup$
    – The_Sympathizer
    Apr 13 at 14:21






  • 1




    $begingroup$
    @TVSuchty : "I see what you did there!" yeah I just made the little bumpzzle perk up :) #mehhr.
    $endgroup$
    – The_Sympathizer
    Apr 13 at 14:45










  • $begingroup$
    @TVSuchty piecewise defined functions are not special and you shouldn't see functions "just as term" as not being piecewise either.
    $endgroup$
    – Tony S.F.
    12 hours ago







1




1




$begingroup$
I see what you did there! Is there a case, where you can write an example without defining the function bit by bit, maybe just as term?
$endgroup$
– TVSuchty
Apr 13 at 9:08




$begingroup$
I see what you did there! Is there a case, where you can write an example without defining the function bit by bit, maybe just as term?
$endgroup$
– TVSuchty
Apr 13 at 9:08




2




2




$begingroup$
@TVSuchty : Actually, yes. See what I just added. I believe this is what you're asking about.
$endgroup$
– The_Sympathizer
Apr 13 at 14:21




$begingroup$
@TVSuchty : Actually, yes. See what I just added. I believe this is what you're asking about.
$endgroup$
– The_Sympathizer
Apr 13 at 14:21




1




1




$begingroup$
@TVSuchty : "I see what you did there!" yeah I just made the little bumpzzle perk up :) #mehhr.
$endgroup$
– The_Sympathizer
Apr 13 at 14:45




$begingroup$
@TVSuchty : "I see what you did there!" yeah I just made the little bumpzzle perk up :) #mehhr.
$endgroup$
– The_Sympathizer
Apr 13 at 14:45












$begingroup$
@TVSuchty piecewise defined functions are not special and you shouldn't see functions "just as term" as not being piecewise either.
$endgroup$
– Tony S.F.
12 hours ago




$begingroup$
@TVSuchty piecewise defined functions are not special and you shouldn't see functions "just as term" as not being piecewise either.
$endgroup$
– Tony S.F.
12 hours ago

















draft saved

draft discarded
















































Thanks for contributing an answer to Mathematics Stack Exchange!


  • Please be sure to answer the question. Provide details and share your research!

But avoid


  • Asking for help, clarification, or responding to other answers.

  • Making statements based on opinion; back them up with references or personal experience.

Use MathJax to format equations. MathJax reference.


To learn more, see our tips on writing great answers.




draft saved


draft discarded














StackExchange.ready(
function ()
StackExchange.openid.initPostLogin('.new-post-login', 'https%3a%2f%2fmath.stackexchange.com%2fquestions%2f3185630%2fare-there-continuous-functions-who-are-the-same-in-an-interval-but-differ-in-at%23new-answer', 'question_page');

);

Post as a guest















Required, but never shown





















































Required, but never shown














Required, but never shown












Required, but never shown







Required, but never shown

































Required, but never shown














Required, but never shown












Required, but never shown







Required, but never shown







Popular posts from this blog

រឿង រ៉ូមេអូ និង ហ្ស៊ុយលីយេ សង្ខេបរឿង តួអង្គ បញ្ជីណែនាំ

QGIS export composer to PDF scale the map [closed] Planned maintenance scheduled April 23, 2019 at 23:30 UTC (7:30pm US/Eastern) Announcing the arrival of Valued Associate #679: Cesar Manara Unicorn Meta Zoo #1: Why another podcast?Print Composer QGIS 2.6, how to export image?QGIS 2.8.1 print composer won't export all OpenCycleMap base layer tilesSave Print/Map QGIS composer view as PNG/PDF using Python (without changing anything in visible layout)?Export QGIS Print Composer PDF with searchable text labelsQGIS Print Composer does not change from landscape to portrait orientation?How can I avoid map size and scale changes in print composer?Fuzzy PDF export in QGIS running on macSierra OSExport the legend into its 100% size using Print ComposerScale-dependent rendering in QGIS PDF output

PDF-ში გადმოწერა სანავიგაციო მენიუproject page